Difference between revisions of "1961 AHSME Problems/Problem 1"

(Created page with ' When simplified <math>(-\frac{1}{125})^{-\frac{2}{3}}</math> becomes:')
(No difference)

Revision as of 16:55, 1 October 2009

When simplified $(-\frac{1}{125})^{-\frac{2}{3}}$ becomes: